Un curieux résultat pour une suite...

Olympiades mathématiques, énigmes et défis
nodjim
Membre Complexe
Messages: 3241
Enregistré le: 24 Avr 2009, 17:35

Un curieux résultat pour une suite...

par nodjim » 12 Aoû 2012, 19:57

Bonsoir à tous,
Cette suite est définie de la manière suivante: u0=1
u(n+1)= 3un si un<1/3 et un/2 dans le cas contraire. Elle détermine donc la comparaison entre les puissances de 3 et les puissances de 2, en quelque sorte. Plus précisément, une puissance de 3 relativement à la puissance de 2 immédiatement supérieure.
Sur un tableur, la recherche des "un" records donne ce résultat (à partir de u4):
u4(1)=0.75
u9(3)=0.84
u14(5)=0.95
u45(17)=0.96
u76(29)=0.97
u107(41)=0.988
u244(94)=0.990
u381(147)=0.992
u518(200)=0.995
u655(253)=0.996
u792(306)=0.998
u2511(971)=0.99902
u4230(1636)=0.99906
u5949(2301)=0.99910
.....
Les nombres entre parenthèses représentent les puissances de 3.
Si on fait l'écart entre ces nombres entre parenthèses, on obtient ce résultat:
2
2
12
12
12
53
53
53
53
53
665
665
665
....
Les records sont battus selon une progression qui ne relève pas du hasard.
Y a t il une explication ?



Doraki
Habitué(e)
Messages: 5021
Enregistré le: 20 Aoû 2008, 12:07

par Doraki » 13 Aoû 2012, 10:15

Pourquoi le record pour u4 c'est pas u4(8000) = 1000 ?

nodjim
Membre Complexe
Messages: 3241
Enregistré le: 24 Avr 2009, 17:35

par nodjim » 13 Aoû 2012, 11:25

C'est à dire que les "un" sont pris dans l'ordre où ils apparaissent. Le record est établi quand on est plus proche de 1 que la valeur déja inscrite. Je démarre à u4 qui donne 1 comme puissance de 3, ça correspond à 3/4. C'est le tableur qui le dit.

Je devine un peu ce qui se passe, si on évite de rester sur les valeurs <1.

Je vais donc dériver quelque peu dans la question:
Soit la suite 3^n des puissances de 3 successives: 3,9,27,...
Prouver qu'il existe une sous suite uk telle que la division par une puissance de 2 judicieusement choisie de 3^n donne des valeurs toujours plus proches de 1.
Par exemple k=1 et k=2 font partie de cette sous suite car pour k=1 3/4 est la valeur la plus proche de 1. Pour k=2, 9/8 est plus proche de 1 que 3/4.

C.Ret
Membre Relatif
Messages: 497
Enregistré le: 02 Juil 2012, 13:33

par C.Ret » 13 Aoû 2012, 13:05

J'ai un souci avec cette suite, je ne trouve pas les valeurs indiquées aux indices donnés:

J'ai peut-être mal compris :


Avec cette définition, je trouve alors les valeurs listée, mais avec un indice décalé:


Quand aux "records", est-ce qu'ils ne correspondent pas tout simplement aux éléments de la suite qui vérifient :
avec k et k' ayant entre-eux une relation binômilae particulière ?

nodjim
Membre Complexe
Messages: 3241
Enregistré le: 24 Avr 2009, 17:35

par nodjim » 13 Aoû 2012, 14:05

OK, C Ret., pour le décalage, ta remarque est pertinente.
Peut être qu'entre k et k' il y a une relation binômiale particulière, je l'ignore. Ce n'est pas la démarche que j'ai suivie. Lire mon second message.

C.Ret
Membre Relatif
Messages: 497
Enregistré le: 02 Juil 2012, 13:33

par C.Ret » 13 Aoû 2012, 15:07

Oui, justement, le raisonnemetn proposé dans ce dernier message me semble une bonne piste pour déterminer cette relation.

En observant la suite u_n pour de très grands nombre, je me rend compte que les "records" n'apparaissent pas aussi régulièrement que je l'escompter. Je suis un peu inquiet, existe-t-il une relation simple ?

nodjim
Membre Complexe
Messages: 3241
Enregistré le: 24 Avr 2009, 17:35

par nodjim » 13 Aoû 2012, 17:45

Peux tu donner un extrait de ce que tu as trouvé pour les "très grands nombres" ?
Il faut se méfier tout de même des capacités de calcul du tableur...
Ma question reste posée.

nodjim
Membre Complexe
Messages: 3241
Enregistré le: 24 Avr 2009, 17:35

par nodjim » 14 Aoû 2012, 09:56

Bon je donne la solution.
F1=1/2=1-1/2
F2=3/2=1+1/2.
Le produit, positif ou négatif, est forcément plus proche de 1 que l'un ou l'autre des facteurs.
F3=F1*F2=1-1/4. Comme F1F4=F2*F3=1+1/8. Comme 1Etc...
On obtient 2 suites, l'une qui donne toujours des valeurs >1, et l'autre des valeurs <1.
Si on remplace Fi par un couple (puissance de 3, puissance de 2), le produit des facteurs correspond à la somme des puissances.
Ainsi, la suite peut se réécrire:
(0,1)-
(1,1)+
(1,2)-
(2,3)+
(3,5)-
(5,8)-
(7,11)+
(12,19)+
...

2 remarques:
Si cette suite atteste qu'on peut se rapprocher de 1 autant que l'on veut, ça ne signifie pas que la différence entre les puissances de 3 et les puissances de 2 se rapprochent, bien au contraire: Grossièrement, on peut estimer le rapport, pour un 3^n, à 1-ou+1/n.

Reste à prouver qu'il n'existe pas de puissances de 3 "hors suite" qui seraient plus proches de 1. Le résultat du tableur indique que non, mais ce n'est pas une preuve.

Doraki
Habitué(e)
Messages: 5021
Enregistré le: 20 Aoû 2008, 12:07

par Doraki » 14 Aoû 2012, 10:12

ben comme ln(3)/ln(2) est irrationnel, ses multiples sont denses dans R/Z, donc tu peux approcher 1 autant que tu veux.

As-tu regardé son développement en fraction continue ? Quand on parle d'approximations rationnelles d'un irrationnel, c'est criminel de ne pas aller voir.
http://www.wolframalpha.com/input/?i=continued+fraction+of+%28ln%283%29%2Fln%282%29%29

Ca doit être vrai que tu obtiens tous les records dans ta liste.
Je sais plus trop comment on le montre, probablement par l'absurde en remontant : Si tu as 3^k dans ta liste qui est juste en-dessous d'une puissance de 2, et si 3^k' est un nouveau record, alors 3^(k'-k) devait être juste au dessus d'une puissance de 2, donc devait probablement être un record (dans l'autre sens) lui-même.

C.Ret
Membre Relatif
Messages: 497
Enregistré le: 02 Juil 2012, 13:33

par C.Ret » 14 Aoû 2012, 14:44

nodjim a écrit:Peux tu donner un extrait de ce que tu as trouvé pour les "très grands nombres" ?



nodjim
Membre Complexe
Messages: 3241
Enregistré le: 24 Avr 2009, 17:35

par nodjim » 15 Aoû 2012, 07:42

Merci C.Ret.
ça correspond exactement à la règle supposée énoncée plus haut. En allant jusqu'aux limites du tableur, ça donne sur la fin:
rapport--------puissances de 3-----écarts
1,000000375--- 9638052--- 190537
1,00000031---- 9828589--- 190537
1,000000246--- 10019126---190537
1,000000181--- 10209663---190537
1,000000117--- 10400200---190537
1,000000052--- 10590737---190537
0,999999988--- 10781274---190537
1,00000004---- 21372011---10590737
1,000000028--- 32153285---10781274
1,000000016--- 42934559---10781274
1,000000003--- 53715833---10781274
0,999999991--- 64497107---10781274
0,999999995--- 118212940---53715833
0,999999998--- 171928773---53715833
1,000000002--- 225644606---53715833
1

Doraki, tu sembles connaitre cette propriété, mais la démo par récurrence ne peut convenir. Ce que je redoute, ce sont des epsilons pas forcément petits, mais opposés et très proches l'un de l'autre:
(1-e1)(1+e2)=1-e1+e2 si on néglige e1*e2.
Or ces cas ne semblent jamais se présenter.

 

Retourner vers ⚔ Défis et énigmes

Qui est en ligne

Utilisateurs parcourant ce forum : Aucun utilisateur enregistré et 16 invités

Tu pars déja ?



Fais toi aider gratuitement sur Maths-forum !

Créé un compte en 1 minute et pose ta question dans le forum ;-)
Inscription gratuite

Identification

Pas encore inscrit ?

Ou identifiez-vous :

Inscription gratuite